Difference between revisions of "2014 AIME I Problems/Problem 8"

(Solution)
(Solution)
Line 1: Line 1:
 
== Problem 8 ==
 
== Problem 8 ==
 
== Solution ==
 
Let <math>x_1< x_2 < x_3</math> be the three real roots of the equation <math>\sqrt{2014} x^3 - 4029x^2 + 2 = 0</math>. Find <math>x_2(x_1+x_3)</math>.
 
 
We note that <math>x=\dfrac{1}{\sqrt{2014}}</math> is a solution since <math>(\dfrac{1}{\sqrt{2014}})^3*\sqrt{2014}-4029(\dfrac{1}{\sqrt{2014}})^2+2=\dfrac{1}{2014}-\dfrac{4029}{2014}+2=\dfrac{1-4029+4028}{2014} = 0</math>
 
 
We claim that <math>x_2=\dfrac{1}{\sqrt{2014}}</math>
 
 
by vieta's formula we have that the <math>x^2</math> coefficent is equal to <math>-x_1-x_2-x_3</math> and that the <math>x</math> coeefficent is equal to <math>x_1x_2+x_1x_3+x_2x_3</math> so using the values in the above equation we get:
 
<math>-x_1-x_2-x_3=-4029\rightarrow x_1+x_2+x_3=4029</math>
 

Revision as of 13:18, 14 March 2014

Problem 8